Hartshorne Problem 1.2.14 on Segre Embedding











up vote
12
down vote

favorite
4












This is a problem in Hartshorne concerning showing that the image of $Bbb{P}^n times Bbb{P}^m$ under the Segre embedding $psi$ is actually irreducible. Now I have shown with some effort that $psi(Bbb{P}^n times Bbb{P}^m)$ is actually equal to $V(mathfrak{a})$ where $mathfrak{a}$ is the ideal generated by the set of all monomials



$$Big{z_{ij}z_{kl} - z_{il}z_{kj} hspace{1mm} Big| hspace{1mm} i,k = 0,ldots, n; hspace{2mm} j,l = 0,ldots,mBig}.$$





My main problem now is in showing that $mathfrak{a}$ is actually equal to the kernel of the ring homomorphism $$varphi : k[z_{ij}] to k[x_0,ldots,x_n,y_0,ldots,y_m]$$
that sends $z_{ij}$ to $x_iy_j$.





I have spent quite a few hours playing around with monomial orderings and trying to show that $mathfrak{a} supseteq ker varphi$ but to no avail. Of course the other inclusion is immediate.



Is there anything I can do apart from playing around with monomial orderings to try and show that the kernel of $varphi$ is equal to $mathfrak{a}$? Perhaps maybe something along the lines of inducting on $n$, those this does not look promising.



Note: Please do not close this question; my question is different from the other questions on this site concerning the Segre embedding. Also, I can show my work concerning how I arrived at the conclusion that $V(mathfrak{a}) = psi(Bbb{P}^n times Bbb{P}^n)$.













share|cite|improve this question
























  • If I remember it correctly, you don't even need to show that $mathfrak{a}=ker(varphi)$, you can prove $V(ker(varphi))=operatorname{im}(psi)$ directly. Could you show us roughly how you proved $V(mathfrak{a})=operatorname{im}(psi)$?
    – Nils Matthes
    Apr 7 '13 at 14:37












  • @NilsMatthes I have posted an answer below.
    – user38268
    Apr 8 '13 at 5:22















up vote
12
down vote

favorite
4












This is a problem in Hartshorne concerning showing that the image of $Bbb{P}^n times Bbb{P}^m$ under the Segre embedding $psi$ is actually irreducible. Now I have shown with some effort that $psi(Bbb{P}^n times Bbb{P}^m)$ is actually equal to $V(mathfrak{a})$ where $mathfrak{a}$ is the ideal generated by the set of all monomials



$$Big{z_{ij}z_{kl} - z_{il}z_{kj} hspace{1mm} Big| hspace{1mm} i,k = 0,ldots, n; hspace{2mm} j,l = 0,ldots,mBig}.$$





My main problem now is in showing that $mathfrak{a}$ is actually equal to the kernel of the ring homomorphism $$varphi : k[z_{ij}] to k[x_0,ldots,x_n,y_0,ldots,y_m]$$
that sends $z_{ij}$ to $x_iy_j$.





I have spent quite a few hours playing around with monomial orderings and trying to show that $mathfrak{a} supseteq ker varphi$ but to no avail. Of course the other inclusion is immediate.



Is there anything I can do apart from playing around with monomial orderings to try and show that the kernel of $varphi$ is equal to $mathfrak{a}$? Perhaps maybe something along the lines of inducting on $n$, those this does not look promising.



Note: Please do not close this question; my question is different from the other questions on this site concerning the Segre embedding. Also, I can show my work concerning how I arrived at the conclusion that $V(mathfrak{a}) = psi(Bbb{P}^n times Bbb{P}^n)$.













share|cite|improve this question
























  • If I remember it correctly, you don't even need to show that $mathfrak{a}=ker(varphi)$, you can prove $V(ker(varphi))=operatorname{im}(psi)$ directly. Could you show us roughly how you proved $V(mathfrak{a})=operatorname{im}(psi)$?
    – Nils Matthes
    Apr 7 '13 at 14:37












  • @NilsMatthes I have posted an answer below.
    – user38268
    Apr 8 '13 at 5:22













up vote
12
down vote

favorite
4









up vote
12
down vote

favorite
4






4





This is a problem in Hartshorne concerning showing that the image of $Bbb{P}^n times Bbb{P}^m$ under the Segre embedding $psi$ is actually irreducible. Now I have shown with some effort that $psi(Bbb{P}^n times Bbb{P}^m)$ is actually equal to $V(mathfrak{a})$ where $mathfrak{a}$ is the ideal generated by the set of all monomials



$$Big{z_{ij}z_{kl} - z_{il}z_{kj} hspace{1mm} Big| hspace{1mm} i,k = 0,ldots, n; hspace{2mm} j,l = 0,ldots,mBig}.$$





My main problem now is in showing that $mathfrak{a}$ is actually equal to the kernel of the ring homomorphism $$varphi : k[z_{ij}] to k[x_0,ldots,x_n,y_0,ldots,y_m]$$
that sends $z_{ij}$ to $x_iy_j$.





I have spent quite a few hours playing around with monomial orderings and trying to show that $mathfrak{a} supseteq ker varphi$ but to no avail. Of course the other inclusion is immediate.



Is there anything I can do apart from playing around with monomial orderings to try and show that the kernel of $varphi$ is equal to $mathfrak{a}$? Perhaps maybe something along the lines of inducting on $n$, those this does not look promising.



Note: Please do not close this question; my question is different from the other questions on this site concerning the Segre embedding. Also, I can show my work concerning how I arrived at the conclusion that $V(mathfrak{a}) = psi(Bbb{P}^n times Bbb{P}^n)$.













share|cite|improve this question















This is a problem in Hartshorne concerning showing that the image of $Bbb{P}^n times Bbb{P}^m$ under the Segre embedding $psi$ is actually irreducible. Now I have shown with some effort that $psi(Bbb{P}^n times Bbb{P}^m)$ is actually equal to $V(mathfrak{a})$ where $mathfrak{a}$ is the ideal generated by the set of all monomials



$$Big{z_{ij}z_{kl} - z_{il}z_{kj} hspace{1mm} Big| hspace{1mm} i,k = 0,ldots, n; hspace{2mm} j,l = 0,ldots,mBig}.$$





My main problem now is in showing that $mathfrak{a}$ is actually equal to the kernel of the ring homomorphism $$varphi : k[z_{ij}] to k[x_0,ldots,x_n,y_0,ldots,y_m]$$
that sends $z_{ij}$ to $x_iy_j$.





I have spent quite a few hours playing around with monomial orderings and trying to show that $mathfrak{a} supseteq ker varphi$ but to no avail. Of course the other inclusion is immediate.



Is there anything I can do apart from playing around with monomial orderings to try and show that the kernel of $varphi$ is equal to $mathfrak{a}$? Perhaps maybe something along the lines of inducting on $n$, those this does not look promising.



Note: Please do not close this question; my question is different from the other questions on this site concerning the Segre embedding. Also, I can show my work concerning how I arrived at the conclusion that $V(mathfrak{a}) = psi(Bbb{P}^n times Bbb{P}^n)$.










algebraic-geometry commutative-algebra invariant-theory algebraic-combinatorics






share|cite|improve this question















share|cite|improve this question













share|cite|improve this question




share|cite|improve this question








edited yesterday









darij grinberg

9,81032961




9,81032961










asked Apr 7 '13 at 13:38







user38268



















  • If I remember it correctly, you don't even need to show that $mathfrak{a}=ker(varphi)$, you can prove $V(ker(varphi))=operatorname{im}(psi)$ directly. Could you show us roughly how you proved $V(mathfrak{a})=operatorname{im}(psi)$?
    – Nils Matthes
    Apr 7 '13 at 14:37












  • @NilsMatthes I have posted an answer below.
    – user38268
    Apr 8 '13 at 5:22


















  • If I remember it correctly, you don't even need to show that $mathfrak{a}=ker(varphi)$, you can prove $V(ker(varphi))=operatorname{im}(psi)$ directly. Could you show us roughly how you proved $V(mathfrak{a})=operatorname{im}(psi)$?
    – Nils Matthes
    Apr 7 '13 at 14:37












  • @NilsMatthes I have posted an answer below.
    – user38268
    Apr 8 '13 at 5:22
















If I remember it correctly, you don't even need to show that $mathfrak{a}=ker(varphi)$, you can prove $V(ker(varphi))=operatorname{im}(psi)$ directly. Could you show us roughly how you proved $V(mathfrak{a})=operatorname{im}(psi)$?
– Nils Matthes
Apr 7 '13 at 14:37






If I remember it correctly, you don't even need to show that $mathfrak{a}=ker(varphi)$, you can prove $V(ker(varphi))=operatorname{im}(psi)$ directly. Could you show us roughly how you proved $V(mathfrak{a})=operatorname{im}(psi)$?
– Nils Matthes
Apr 7 '13 at 14:37














@NilsMatthes I have posted an answer below.
– user38268
Apr 8 '13 at 5:22




@NilsMatthes I have posted an answer below.
– user38268
Apr 8 '13 at 5:22










3 Answers
3






active

oldest

votes

















up vote
9
down vote













I outlined a proof by Bjorn Poonen at http://www.artofproblemsolving.com/Forum/viewtopic.php?p=2450856#p2450856 . As Nils Matthes has noticed, computing the kernel is not necessary to solve Hartshorne's problem, though (in my opinion) it is more interesting than the problem itself.





$newcommand{kk}{mathbb{k}}$
$newcommand{Ker}{operatorname{Ker}}$
For the sake of self-containedness, let me repost the proof linked above here (with improved notations). I begin by restating the problem:




Theorem 1. Let $kk$ be a commutative ring with $1$. Let $n$ and $m$ be two nonnegative integers. Let $M=left{0,1,ldots ,mright}$ and $N=left{0,1,ldots ,nright}$.



Let $R$ be the polynomial ring $kkleft[Z_{i,j} mid i in M text{ and } j in N right]$.



Let $S$ be the polynomial ring $kkleft[x_0, x_1, ldots, x_m, y_0, y_1, ldots, y_n right]$.



Let $phi : R to S$ be the unique $kk$-algebra homomorphism that sends each $Z_{i,j}$ to $x_i y_j$.



Let $W$ be the ideal of $R$ generated by all elements of the type $Z_{a,b} Z_{c,d} - Z_{a,d} Z_{c,b}$ with $a in M$, $b in N$, $c in M$ and $d in N$.



Then, $Ker phi = W$.




To prove this, we shall use the following easy algebraic lemma:




Lemma 2. Let $C$ be a $kk$-module. Let $A$ and $B$ be two submodules of $C$ such that $C=A+B$. Let $psi$ be a $kk$-module map from $C$ to another $kk$-module $D$ such that $psimid_A$ is injective and $Ker psi supseteq B$. Then, $Ker psi = B$.




Proof of Lemma 2. Let $c in Ker psi$. Thus, $c in Ker psi subseteq C = A + B$; hence, we can write $c$ in the form $c = a + b$ for some $a in A$ and $b in B$. Consider these $a$ and $b$. We have $b in B subseteq Ker psi$, so that $psileft(bright) = 0$. Applying the map $psi$ to the equality $c = a + b$, we obtain $psileft(cright) = psileft(a + bright) = psileft(aright) + psileft(bright)$ (since $psi$ is a $kk$-module map). Comparing this with $psileft(cright) = 0$ (which follows from $c in Ker psi$), we obtain $0 = psileft(aright) + underbrace{psileft(bright)}_{= 0} = psileft(aright)$, so that $psileft(aright) = 0 = psileft(0right)$. Since $psimid_A$ is injective, this entails $a = 0$ (because both $a$ and $0$ belong to $A$). Thus, $c = underbrace{a}_{=0} + b = b in B$.



Now, forget that we fixed $c$. We thus have shown that $c in B$ for each $c in Ker psi$. Thus, $Ker psi subseteq B$. Combining this with $Ker psi supseteq B$, we obtain $Ker psi = B$. This proves Lemma 2. $blacksquare$



Proof of Theorem 1 (Bjorn Poonen) (sketched). We notice that $Ker phisupseteq W$ is very easy to prove (in fact, a trivial computation shows that $Ker phi$ contains $Z_{a,b}Z_{c,d}-Z_{a,d}Z_{c,b}$ for all $a$, $b$, $c$, $d$).



We order the set $Mtimes N$ lexicographically.



If $k$ is a nonnegative integer, then $S_k$ shall denote the symmetric group consisting of all permutations of $left{1,2,ldots,kright}$.
Every $k$-tuple $left(left(a_1,b_1right),left(a_2,b_2right),ldots ,left(a_k,b_kright)right) in left(Mtimes Nright)^k$ and every permutation $sigma in S_k$ satisfy



begin{equation}
Z_{a_1,b_1}Z_{a_2,b_2}cdots Z_{a_k,b_k} equiv Z_{a_1,b_{sigma 1}}Z_{a_2,b_{sigma 2}}cdots Z_{a_k,b_{sigma k}} mod W .
label{darij.pf.thm1.1}
tag{1}
end{equation}



(In fact, this is obvious from the definition of $W$ when $sigma$ is a transposition, and hence, by induction, it also holds for every permutation $sigma$, because every permutation is a composition of transpositions.)



Now, let $T$ be the $kk$-submodule of $kkleft[Z_{i,j}mid left(i,jright)in Mtimes Nright]$ generated by all products of the form $Z_{a_1,c_1}Z_{a_2,c_2}cdots Z_{a_k,c_k}$ with $k$ being a nonnegative integer and $left(left(a_1,c_1right),left(a_2,c_2right),ldots,left(a_k,c_kright)right)in left(Mtimes Nright)^k$ being a $k$-tuple satisfying $a_1leq a_2leq cdotsleq a_k$ and $c_1leq c_2leq cdotsleq c_k$. It is easy to see that the map $left.phimid_Tright. : T to kkleft[x_0,x_1,ldots,x_m,y_0,y_1,ldots,y_nright]$ is injective. (In fact, if $left(left(a_1,c_1right),left(a_2,c_2right),ldots,left(a_k,c_kright)right)in T$, then
begin{align}
left(phimid_Tright)left(Z_{a_1,c_1}Z_{a_2,c_2}cdots Z_{a_k,c_k}right)
&= phileft(Z_{a_1,c_1}Z_{a_2,c_2}cdots Z_{a_k,c_k}right) \
&= x_{a_1}y_{c_1}x_{a_2}y_{c_2}cdots x_{a_k}y_{c_k} \
&=x_{a_1}x_{a_2}cdots x_{a_k}y_{c_1}y_{c_2}cdots y_{c_k}
end{align}

is a monomial from which we can recover the $k$-tuple $left(a_1,a_2,ldots ,a_kright)$ up to order and the $k$-tuple $left(c_1,c_2,ldots ,c_kright)$ up to order; but since the order of each of these two $k$-tuples is predetermined by the condition that $a_1leq a_2leq cdotsleq a_k$ and $c_1leq c_2leq cdotsleq c_k$, we can therefore recover these two $k$-tuples completely; hence, the map $phimid_T$ sends distinct monomials to distinct monomials, and thus is injective.)



Next we are going to show that:



begin{equation}
text{every monomial in $kkleft[Z_{i,j}mid left(i,jright)in Mtimes Nright]$ lies in $T+W$.}
label{darij.pf.thm1.2}
tag{2}
end{equation}



[Proof of eqref{darij.pf.thm1.2}: Let $mu$ be any monomial in $kkleft[Z_{i,j}mid left(i,jright)in Mtimes Nright]$. Then, $mu=Z_{a_1,b_1}Z_{a_2,b_2}cdots Z_{a_k,b_k}$ for some nonnegative integer $k$ and some $k$-tuple $left(left(a_1,b_1right),left(a_2,b_2right),ldots,left(a_k,b_kright)right)in left(Mtimes Nright)^k$ such that $left(a_1,b_1right)leqleft(a_2,b_2right)leq cdotsleq left(a_k,b_kright)$. Consider such a $k$ and such a $k$-tuple $left(left(a_1,b_1right),left(a_2,b_2right),ldots,left(a_k,b_kright)right)$. Since $left(a_1,b_1right)leqleft(a_2,b_2right)leq cdotsleq left(a_k,b_kright)$, we have $a_1leq a_2leq cdotsleq a_k$ (since our order is lexicographic). Clearly there exists a permutation $sigmain S_k$ such that $b_{sigma 1}leq b_{sigma 2}leq cdots leq b_{sigma k}$. Consider such a $sigma$. Let $c_i=b_{sigma i}$ for every $iinleft{1,2,ldots,kright}$. Hence, the chain of inequalities $b_{sigma 1}leq b_{sigma 2}leq cdots leq b_{sigma k}$ rewrites as $c_1leq c_2leq cdotsleq c_k$. Also,
begin{align}
mu
&= Z_{a_1,b_1}Z_{a_2,b_2}cdots Z_{a_k,b_k} \
&equiv Z_{a_1,b_{sigma 1}}Z_{a_2,b_{sigma 2}}cdots Z_{a_k,b_{sigma k}}
qquad left( text{by eqref{darij.pf.thm1.1}} right) \
&= Z_{a_1,c_1} Z_{a_2,c_2} cdots Z_{a_k,c_k} mod W
end{align}

(since $b_{sigma i}=c_i$ for every $iinleft{1,2,ldots,kright}$).



But since $a_1leq a_2leq cdotsleq a_k$ and $c_1leq c_2leq cdotsleq c_k$, we have $Z_{a_1,c_1} Z_{a_2,c_2} cdots Z_{a_k,c_k}in T$ (by the definition of $T$), so this rewrites as follows:
begin{align}
mu
&equiv left(text{an element of }Tright)mod W .
end{align}

In other words, $muin T+W$. Since this holds for every monomial $mu$ in $kkleft[Z_{i,j}mid left(i,jright)in Mtimes Nright]$, this proves eqref{darij.pf.thm1.2}.]



Since the monomials in $kkleft[Z_{i,j}mid left(i,jright)in Mtimes Nright]$ generate the $kk$-module $kkleft[Z_{i,j}mid left(i,jright)in Mtimes Nright]$, and since $T+W$ is a submodule of this $kk$-module, we obtain $kkleft[Z_{i,j}mid left(i,jright)in Mtimes Nright]=T+W$ from eqref{darij.pf.thm1.2}.



Applying Lemma 2 to $C=kkleft[Z_{i,j}mid left(i,jright)in Mtimes Nright]$, $A=T$, $B=W$ and $psi=phi$, we thus conclude that $Ker phi = W$. This proves Theorem 1. $blacksquare$





There is yet another way to prove Theorem 1 -- namely, by revealing it to be a particular case of the Second Fundamental Theorem of Invariant Theory for GL. See https://mathoverflow.net/questions/202005/a-vector-version-of-the-segre-embedding-what-is-the-kernel-of-the-ring-map for this generalization. (Another place where this generalization appears with proof is Theorem 5.1 of J. Désarménien, Joseph P. S. Kung, Gian-Carlo Rota, Invariant Theory, Young Bitableaux, and Combinatorics, unofficial re-edition 2017; you just need to set $d = 1$, and realize that every standard $left(mathcal{X},mathcal{U}right)$-bideterminant of shape strictly longer than $left(dright)$ contains at least one row of length $geq 2$, which is easily seen to place it inside the ideal $W$.)






share|cite|improve this answer



















  • 3




    Dear Darij, thanks for your answer. I posted an answer below showing that we don't need to prove explicitly that the kernel is a prime ideal to show that the image of the Segre embedding is a projective variety. It should be correct. Regards,
    – user38268
    Apr 8 '13 at 5:55










  • Regarding OP's original solution - why is the observation that $operatorname{Im}psi$ equals the vanishing locus of the ideal $I$ generated by the mentioned monomials, along with the observation $I$ is prime not enough?
    – Arrow
    2 days ago










  • @Arrow: how do you prove $I$ is prime?
    – darij grinberg
    2 days ago










  • thanks for this. It's great! BTW, in your definition of $T$ don't you need to use strict inequalities?
    – user347489
    yesterday










  • @user347489: No, I don't. Is anything wrong with the argument?
    – darij grinberg
    yesterday


















up vote
6
down vote



accepted










Indeed as Nils Matthes suggested we don't know need to go through such a mess and just use the hint of Hartshorne.



Define a map $varphi : k[T_{00} , ldots, T_{nm}] to k[X_0,ldots,X_n,Y_0,ldots,Y_m]$ that sends $T_{ij}$ to $X_iY_j$. Let $mathfrak{a} := ker varphi$. We claim that $psi (Bbb{P}^n times Bbb{P}^m) = V(mathfrak{a})$. For one inclusion if a point
$$a = [a_{00} : ldots : a_{nm}] in V(mathfrak{a})$$
then in particular $a$ is a zero of all the polynomials $T_{ij}T_{kl} - T_{il}T_{kj}$. But this means that $a in psi(Bbb{P}^n times Bbb{P}^m)$. The reverse inclusion follows immediately from the definition of $varphi$. Thus $V(mathfrak{a}) = psi(Bbb{P}^n times Bbb{P}^m)$ and the projective Nullstellensatz implies that $psi(Bbb{P}^n times Bbb{P}^m)$ is a projective variety.






share|cite|improve this answer



















  • 1




    Shouldn't you prove that $mathfrak a$ is actually a homogeneous ideal?
    – Andrea Gagna
    May 15 '14 at 15:46


















up vote
4
down vote













It's easy to show the preimage under the Segre map of an algebraic set is an algebraic subset of $mathbb{P}^n times mathbb{P}^m.$ If $V$ were reducible, we could write $V = V_1 cup V_2,$ where $V_ineq V$ are closed. Then $$ mathbb{P}^n times mathbb{P}^m = S^{-1}(V) = S^{-1} (V_1) bigcup S^{-1}(V_2)$$ where $S^{-1}(V_i)$ are closed. Picking $x_i in Vsetminus V_i$ we have $S^{-1}(x_i)cap S^{-1}(V_i)=emptyset$ so $S^{-1}(V_i) subset S^{-1}(V)$ is a strict inclusion, contradicting that $mathbb{P}^n times mathbb{P}^m$ is irreducible.






share|cite|improve this answer

















  • 3




    Of course we do, it's the same condition as always in any topological space: A space $X$ is reducible if $X= X_1 cup X_2$ for some closed proper subsets $X_1, X_2$ and irreducible if not reducible. A subset $V$ of $mathbb{P}^n times mathbb{P}^m$ is closed if $V=V(S)$ where $S$ is a set of biforms in $k[X_0, cdots, X_n, Y_0, cdots, Y_m].$ This already establishes what it means for $mathbb{P}^n times mathbb{P}^m$ to be irreducible.
    – Ragib Zaman
    Jun 3 '13 at 10:11











Your Answer





StackExchange.ifUsing("editor", function () {
return StackExchange.using("mathjaxEditing", function () {
StackExchange.MarkdownEditor.creationCallbacks.add(function (editor, postfix) {
StackExchange.mathjaxEditing.prepareWmdForMathJax(editor, postfix, [["$", "$"], ["\\(","\\)"]]);
});
});
}, "mathjax-editing");

StackExchange.ready(function() {
var channelOptions = {
tags: "".split(" "),
id: "69"
};
initTagRenderer("".split(" "), "".split(" "), channelOptions);

StackExchange.using("externalEditor", function() {
// Have to fire editor after snippets, if snippets enabled
if (StackExchange.settings.snippets.snippetsEnabled) {
StackExchange.using("snippets", function() {
createEditor();
});
}
else {
createEditor();
}
});

function createEditor() {
StackExchange.prepareEditor({
heartbeatType: 'answer',
convertImagesToLinks: true,
noModals: true,
showLowRepImageUploadWarning: true,
reputationToPostImages: 10,
bindNavPrevention: true,
postfix: "",
imageUploader: {
brandingHtml: "Powered by u003ca class="icon-imgur-white" href="https://imgur.com/"u003eu003c/au003e",
contentPolicyHtml: "User contributions licensed under u003ca href="https://creativecommons.org/licenses/by-sa/3.0/"u003ecc by-sa 3.0 with attribution requiredu003c/au003e u003ca href="https://stackoverflow.com/legal/content-policy"u003e(content policy)u003c/au003e",
allowUrls: true
},
noCode: true, onDemand: true,
discardSelector: ".discard-answer"
,immediatelyShowMarkdownHelp:true
});


}
});














 

draft saved


draft discarded


















StackExchange.ready(
function () {
StackExchange.openid.initPostLogin('.new-post-login', 'https%3a%2f%2fmath.stackexchange.com%2fquestions%2f353846%2fhartshorne-problem-1-2-14-on-segre-embedding%23new-answer', 'question_page');
}
);

Post as a guest















Required, but never shown
























3 Answers
3






active

oldest

votes








3 Answers
3






active

oldest

votes









active

oldest

votes






active

oldest

votes








up vote
9
down vote













I outlined a proof by Bjorn Poonen at http://www.artofproblemsolving.com/Forum/viewtopic.php?p=2450856#p2450856 . As Nils Matthes has noticed, computing the kernel is not necessary to solve Hartshorne's problem, though (in my opinion) it is more interesting than the problem itself.





$newcommand{kk}{mathbb{k}}$
$newcommand{Ker}{operatorname{Ker}}$
For the sake of self-containedness, let me repost the proof linked above here (with improved notations). I begin by restating the problem:




Theorem 1. Let $kk$ be a commutative ring with $1$. Let $n$ and $m$ be two nonnegative integers. Let $M=left{0,1,ldots ,mright}$ and $N=left{0,1,ldots ,nright}$.



Let $R$ be the polynomial ring $kkleft[Z_{i,j} mid i in M text{ and } j in N right]$.



Let $S$ be the polynomial ring $kkleft[x_0, x_1, ldots, x_m, y_0, y_1, ldots, y_n right]$.



Let $phi : R to S$ be the unique $kk$-algebra homomorphism that sends each $Z_{i,j}$ to $x_i y_j$.



Let $W$ be the ideal of $R$ generated by all elements of the type $Z_{a,b} Z_{c,d} - Z_{a,d} Z_{c,b}$ with $a in M$, $b in N$, $c in M$ and $d in N$.



Then, $Ker phi = W$.




To prove this, we shall use the following easy algebraic lemma:




Lemma 2. Let $C$ be a $kk$-module. Let $A$ and $B$ be two submodules of $C$ such that $C=A+B$. Let $psi$ be a $kk$-module map from $C$ to another $kk$-module $D$ such that $psimid_A$ is injective and $Ker psi supseteq B$. Then, $Ker psi = B$.




Proof of Lemma 2. Let $c in Ker psi$. Thus, $c in Ker psi subseteq C = A + B$; hence, we can write $c$ in the form $c = a + b$ for some $a in A$ and $b in B$. Consider these $a$ and $b$. We have $b in B subseteq Ker psi$, so that $psileft(bright) = 0$. Applying the map $psi$ to the equality $c = a + b$, we obtain $psileft(cright) = psileft(a + bright) = psileft(aright) + psileft(bright)$ (since $psi$ is a $kk$-module map). Comparing this with $psileft(cright) = 0$ (which follows from $c in Ker psi$), we obtain $0 = psileft(aright) + underbrace{psileft(bright)}_{= 0} = psileft(aright)$, so that $psileft(aright) = 0 = psileft(0right)$. Since $psimid_A$ is injective, this entails $a = 0$ (because both $a$ and $0$ belong to $A$). Thus, $c = underbrace{a}_{=0} + b = b in B$.



Now, forget that we fixed $c$. We thus have shown that $c in B$ for each $c in Ker psi$. Thus, $Ker psi subseteq B$. Combining this with $Ker psi supseteq B$, we obtain $Ker psi = B$. This proves Lemma 2. $blacksquare$



Proof of Theorem 1 (Bjorn Poonen) (sketched). We notice that $Ker phisupseteq W$ is very easy to prove (in fact, a trivial computation shows that $Ker phi$ contains $Z_{a,b}Z_{c,d}-Z_{a,d}Z_{c,b}$ for all $a$, $b$, $c$, $d$).



We order the set $Mtimes N$ lexicographically.



If $k$ is a nonnegative integer, then $S_k$ shall denote the symmetric group consisting of all permutations of $left{1,2,ldots,kright}$.
Every $k$-tuple $left(left(a_1,b_1right),left(a_2,b_2right),ldots ,left(a_k,b_kright)right) in left(Mtimes Nright)^k$ and every permutation $sigma in S_k$ satisfy



begin{equation}
Z_{a_1,b_1}Z_{a_2,b_2}cdots Z_{a_k,b_k} equiv Z_{a_1,b_{sigma 1}}Z_{a_2,b_{sigma 2}}cdots Z_{a_k,b_{sigma k}} mod W .
label{darij.pf.thm1.1}
tag{1}
end{equation}



(In fact, this is obvious from the definition of $W$ when $sigma$ is a transposition, and hence, by induction, it also holds for every permutation $sigma$, because every permutation is a composition of transpositions.)



Now, let $T$ be the $kk$-submodule of $kkleft[Z_{i,j}mid left(i,jright)in Mtimes Nright]$ generated by all products of the form $Z_{a_1,c_1}Z_{a_2,c_2}cdots Z_{a_k,c_k}$ with $k$ being a nonnegative integer and $left(left(a_1,c_1right),left(a_2,c_2right),ldots,left(a_k,c_kright)right)in left(Mtimes Nright)^k$ being a $k$-tuple satisfying $a_1leq a_2leq cdotsleq a_k$ and $c_1leq c_2leq cdotsleq c_k$. It is easy to see that the map $left.phimid_Tright. : T to kkleft[x_0,x_1,ldots,x_m,y_0,y_1,ldots,y_nright]$ is injective. (In fact, if $left(left(a_1,c_1right),left(a_2,c_2right),ldots,left(a_k,c_kright)right)in T$, then
begin{align}
left(phimid_Tright)left(Z_{a_1,c_1}Z_{a_2,c_2}cdots Z_{a_k,c_k}right)
&= phileft(Z_{a_1,c_1}Z_{a_2,c_2}cdots Z_{a_k,c_k}right) \
&= x_{a_1}y_{c_1}x_{a_2}y_{c_2}cdots x_{a_k}y_{c_k} \
&=x_{a_1}x_{a_2}cdots x_{a_k}y_{c_1}y_{c_2}cdots y_{c_k}
end{align}

is a monomial from which we can recover the $k$-tuple $left(a_1,a_2,ldots ,a_kright)$ up to order and the $k$-tuple $left(c_1,c_2,ldots ,c_kright)$ up to order; but since the order of each of these two $k$-tuples is predetermined by the condition that $a_1leq a_2leq cdotsleq a_k$ and $c_1leq c_2leq cdotsleq c_k$, we can therefore recover these two $k$-tuples completely; hence, the map $phimid_T$ sends distinct monomials to distinct monomials, and thus is injective.)



Next we are going to show that:



begin{equation}
text{every monomial in $kkleft[Z_{i,j}mid left(i,jright)in Mtimes Nright]$ lies in $T+W$.}
label{darij.pf.thm1.2}
tag{2}
end{equation}



[Proof of eqref{darij.pf.thm1.2}: Let $mu$ be any monomial in $kkleft[Z_{i,j}mid left(i,jright)in Mtimes Nright]$. Then, $mu=Z_{a_1,b_1}Z_{a_2,b_2}cdots Z_{a_k,b_k}$ for some nonnegative integer $k$ and some $k$-tuple $left(left(a_1,b_1right),left(a_2,b_2right),ldots,left(a_k,b_kright)right)in left(Mtimes Nright)^k$ such that $left(a_1,b_1right)leqleft(a_2,b_2right)leq cdotsleq left(a_k,b_kright)$. Consider such a $k$ and such a $k$-tuple $left(left(a_1,b_1right),left(a_2,b_2right),ldots,left(a_k,b_kright)right)$. Since $left(a_1,b_1right)leqleft(a_2,b_2right)leq cdotsleq left(a_k,b_kright)$, we have $a_1leq a_2leq cdotsleq a_k$ (since our order is lexicographic). Clearly there exists a permutation $sigmain S_k$ such that $b_{sigma 1}leq b_{sigma 2}leq cdots leq b_{sigma k}$. Consider such a $sigma$. Let $c_i=b_{sigma i}$ for every $iinleft{1,2,ldots,kright}$. Hence, the chain of inequalities $b_{sigma 1}leq b_{sigma 2}leq cdots leq b_{sigma k}$ rewrites as $c_1leq c_2leq cdotsleq c_k$. Also,
begin{align}
mu
&= Z_{a_1,b_1}Z_{a_2,b_2}cdots Z_{a_k,b_k} \
&equiv Z_{a_1,b_{sigma 1}}Z_{a_2,b_{sigma 2}}cdots Z_{a_k,b_{sigma k}}
qquad left( text{by eqref{darij.pf.thm1.1}} right) \
&= Z_{a_1,c_1} Z_{a_2,c_2} cdots Z_{a_k,c_k} mod W
end{align}

(since $b_{sigma i}=c_i$ for every $iinleft{1,2,ldots,kright}$).



But since $a_1leq a_2leq cdotsleq a_k$ and $c_1leq c_2leq cdotsleq c_k$, we have $Z_{a_1,c_1} Z_{a_2,c_2} cdots Z_{a_k,c_k}in T$ (by the definition of $T$), so this rewrites as follows:
begin{align}
mu
&equiv left(text{an element of }Tright)mod W .
end{align}

In other words, $muin T+W$. Since this holds for every monomial $mu$ in $kkleft[Z_{i,j}mid left(i,jright)in Mtimes Nright]$, this proves eqref{darij.pf.thm1.2}.]



Since the monomials in $kkleft[Z_{i,j}mid left(i,jright)in Mtimes Nright]$ generate the $kk$-module $kkleft[Z_{i,j}mid left(i,jright)in Mtimes Nright]$, and since $T+W$ is a submodule of this $kk$-module, we obtain $kkleft[Z_{i,j}mid left(i,jright)in Mtimes Nright]=T+W$ from eqref{darij.pf.thm1.2}.



Applying Lemma 2 to $C=kkleft[Z_{i,j}mid left(i,jright)in Mtimes Nright]$, $A=T$, $B=W$ and $psi=phi$, we thus conclude that $Ker phi = W$. This proves Theorem 1. $blacksquare$





There is yet another way to prove Theorem 1 -- namely, by revealing it to be a particular case of the Second Fundamental Theorem of Invariant Theory for GL. See https://mathoverflow.net/questions/202005/a-vector-version-of-the-segre-embedding-what-is-the-kernel-of-the-ring-map for this generalization. (Another place where this generalization appears with proof is Theorem 5.1 of J. Désarménien, Joseph P. S. Kung, Gian-Carlo Rota, Invariant Theory, Young Bitableaux, and Combinatorics, unofficial re-edition 2017; you just need to set $d = 1$, and realize that every standard $left(mathcal{X},mathcal{U}right)$-bideterminant of shape strictly longer than $left(dright)$ contains at least one row of length $geq 2$, which is easily seen to place it inside the ideal $W$.)






share|cite|improve this answer



















  • 3




    Dear Darij, thanks for your answer. I posted an answer below showing that we don't need to prove explicitly that the kernel is a prime ideal to show that the image of the Segre embedding is a projective variety. It should be correct. Regards,
    – user38268
    Apr 8 '13 at 5:55










  • Regarding OP's original solution - why is the observation that $operatorname{Im}psi$ equals the vanishing locus of the ideal $I$ generated by the mentioned monomials, along with the observation $I$ is prime not enough?
    – Arrow
    2 days ago










  • @Arrow: how do you prove $I$ is prime?
    – darij grinberg
    2 days ago










  • thanks for this. It's great! BTW, in your definition of $T$ don't you need to use strict inequalities?
    – user347489
    yesterday










  • @user347489: No, I don't. Is anything wrong with the argument?
    – darij grinberg
    yesterday















up vote
9
down vote













I outlined a proof by Bjorn Poonen at http://www.artofproblemsolving.com/Forum/viewtopic.php?p=2450856#p2450856 . As Nils Matthes has noticed, computing the kernel is not necessary to solve Hartshorne's problem, though (in my opinion) it is more interesting than the problem itself.





$newcommand{kk}{mathbb{k}}$
$newcommand{Ker}{operatorname{Ker}}$
For the sake of self-containedness, let me repost the proof linked above here (with improved notations). I begin by restating the problem:




Theorem 1. Let $kk$ be a commutative ring with $1$. Let $n$ and $m$ be two nonnegative integers. Let $M=left{0,1,ldots ,mright}$ and $N=left{0,1,ldots ,nright}$.



Let $R$ be the polynomial ring $kkleft[Z_{i,j} mid i in M text{ and } j in N right]$.



Let $S$ be the polynomial ring $kkleft[x_0, x_1, ldots, x_m, y_0, y_1, ldots, y_n right]$.



Let $phi : R to S$ be the unique $kk$-algebra homomorphism that sends each $Z_{i,j}$ to $x_i y_j$.



Let $W$ be the ideal of $R$ generated by all elements of the type $Z_{a,b} Z_{c,d} - Z_{a,d} Z_{c,b}$ with $a in M$, $b in N$, $c in M$ and $d in N$.



Then, $Ker phi = W$.




To prove this, we shall use the following easy algebraic lemma:




Lemma 2. Let $C$ be a $kk$-module. Let $A$ and $B$ be two submodules of $C$ such that $C=A+B$. Let $psi$ be a $kk$-module map from $C$ to another $kk$-module $D$ such that $psimid_A$ is injective and $Ker psi supseteq B$. Then, $Ker psi = B$.




Proof of Lemma 2. Let $c in Ker psi$. Thus, $c in Ker psi subseteq C = A + B$; hence, we can write $c$ in the form $c = a + b$ for some $a in A$ and $b in B$. Consider these $a$ and $b$. We have $b in B subseteq Ker psi$, so that $psileft(bright) = 0$. Applying the map $psi$ to the equality $c = a + b$, we obtain $psileft(cright) = psileft(a + bright) = psileft(aright) + psileft(bright)$ (since $psi$ is a $kk$-module map). Comparing this with $psileft(cright) = 0$ (which follows from $c in Ker psi$), we obtain $0 = psileft(aright) + underbrace{psileft(bright)}_{= 0} = psileft(aright)$, so that $psileft(aright) = 0 = psileft(0right)$. Since $psimid_A$ is injective, this entails $a = 0$ (because both $a$ and $0$ belong to $A$). Thus, $c = underbrace{a}_{=0} + b = b in B$.



Now, forget that we fixed $c$. We thus have shown that $c in B$ for each $c in Ker psi$. Thus, $Ker psi subseteq B$. Combining this with $Ker psi supseteq B$, we obtain $Ker psi = B$. This proves Lemma 2. $blacksquare$



Proof of Theorem 1 (Bjorn Poonen) (sketched). We notice that $Ker phisupseteq W$ is very easy to prove (in fact, a trivial computation shows that $Ker phi$ contains $Z_{a,b}Z_{c,d}-Z_{a,d}Z_{c,b}$ for all $a$, $b$, $c$, $d$).



We order the set $Mtimes N$ lexicographically.



If $k$ is a nonnegative integer, then $S_k$ shall denote the symmetric group consisting of all permutations of $left{1,2,ldots,kright}$.
Every $k$-tuple $left(left(a_1,b_1right),left(a_2,b_2right),ldots ,left(a_k,b_kright)right) in left(Mtimes Nright)^k$ and every permutation $sigma in S_k$ satisfy



begin{equation}
Z_{a_1,b_1}Z_{a_2,b_2}cdots Z_{a_k,b_k} equiv Z_{a_1,b_{sigma 1}}Z_{a_2,b_{sigma 2}}cdots Z_{a_k,b_{sigma k}} mod W .
label{darij.pf.thm1.1}
tag{1}
end{equation}



(In fact, this is obvious from the definition of $W$ when $sigma$ is a transposition, and hence, by induction, it also holds for every permutation $sigma$, because every permutation is a composition of transpositions.)



Now, let $T$ be the $kk$-submodule of $kkleft[Z_{i,j}mid left(i,jright)in Mtimes Nright]$ generated by all products of the form $Z_{a_1,c_1}Z_{a_2,c_2}cdots Z_{a_k,c_k}$ with $k$ being a nonnegative integer and $left(left(a_1,c_1right),left(a_2,c_2right),ldots,left(a_k,c_kright)right)in left(Mtimes Nright)^k$ being a $k$-tuple satisfying $a_1leq a_2leq cdotsleq a_k$ and $c_1leq c_2leq cdotsleq c_k$. It is easy to see that the map $left.phimid_Tright. : T to kkleft[x_0,x_1,ldots,x_m,y_0,y_1,ldots,y_nright]$ is injective. (In fact, if $left(left(a_1,c_1right),left(a_2,c_2right),ldots,left(a_k,c_kright)right)in T$, then
begin{align}
left(phimid_Tright)left(Z_{a_1,c_1}Z_{a_2,c_2}cdots Z_{a_k,c_k}right)
&= phileft(Z_{a_1,c_1}Z_{a_2,c_2}cdots Z_{a_k,c_k}right) \
&= x_{a_1}y_{c_1}x_{a_2}y_{c_2}cdots x_{a_k}y_{c_k} \
&=x_{a_1}x_{a_2}cdots x_{a_k}y_{c_1}y_{c_2}cdots y_{c_k}
end{align}

is a monomial from which we can recover the $k$-tuple $left(a_1,a_2,ldots ,a_kright)$ up to order and the $k$-tuple $left(c_1,c_2,ldots ,c_kright)$ up to order; but since the order of each of these two $k$-tuples is predetermined by the condition that $a_1leq a_2leq cdotsleq a_k$ and $c_1leq c_2leq cdotsleq c_k$, we can therefore recover these two $k$-tuples completely; hence, the map $phimid_T$ sends distinct monomials to distinct monomials, and thus is injective.)



Next we are going to show that:



begin{equation}
text{every monomial in $kkleft[Z_{i,j}mid left(i,jright)in Mtimes Nright]$ lies in $T+W$.}
label{darij.pf.thm1.2}
tag{2}
end{equation}



[Proof of eqref{darij.pf.thm1.2}: Let $mu$ be any monomial in $kkleft[Z_{i,j}mid left(i,jright)in Mtimes Nright]$. Then, $mu=Z_{a_1,b_1}Z_{a_2,b_2}cdots Z_{a_k,b_k}$ for some nonnegative integer $k$ and some $k$-tuple $left(left(a_1,b_1right),left(a_2,b_2right),ldots,left(a_k,b_kright)right)in left(Mtimes Nright)^k$ such that $left(a_1,b_1right)leqleft(a_2,b_2right)leq cdotsleq left(a_k,b_kright)$. Consider such a $k$ and such a $k$-tuple $left(left(a_1,b_1right),left(a_2,b_2right),ldots,left(a_k,b_kright)right)$. Since $left(a_1,b_1right)leqleft(a_2,b_2right)leq cdotsleq left(a_k,b_kright)$, we have $a_1leq a_2leq cdotsleq a_k$ (since our order is lexicographic). Clearly there exists a permutation $sigmain S_k$ such that $b_{sigma 1}leq b_{sigma 2}leq cdots leq b_{sigma k}$. Consider such a $sigma$. Let $c_i=b_{sigma i}$ for every $iinleft{1,2,ldots,kright}$. Hence, the chain of inequalities $b_{sigma 1}leq b_{sigma 2}leq cdots leq b_{sigma k}$ rewrites as $c_1leq c_2leq cdotsleq c_k$. Also,
begin{align}
mu
&= Z_{a_1,b_1}Z_{a_2,b_2}cdots Z_{a_k,b_k} \
&equiv Z_{a_1,b_{sigma 1}}Z_{a_2,b_{sigma 2}}cdots Z_{a_k,b_{sigma k}}
qquad left( text{by eqref{darij.pf.thm1.1}} right) \
&= Z_{a_1,c_1} Z_{a_2,c_2} cdots Z_{a_k,c_k} mod W
end{align}

(since $b_{sigma i}=c_i$ for every $iinleft{1,2,ldots,kright}$).



But since $a_1leq a_2leq cdotsleq a_k$ and $c_1leq c_2leq cdotsleq c_k$, we have $Z_{a_1,c_1} Z_{a_2,c_2} cdots Z_{a_k,c_k}in T$ (by the definition of $T$), so this rewrites as follows:
begin{align}
mu
&equiv left(text{an element of }Tright)mod W .
end{align}

In other words, $muin T+W$. Since this holds for every monomial $mu$ in $kkleft[Z_{i,j}mid left(i,jright)in Mtimes Nright]$, this proves eqref{darij.pf.thm1.2}.]



Since the monomials in $kkleft[Z_{i,j}mid left(i,jright)in Mtimes Nright]$ generate the $kk$-module $kkleft[Z_{i,j}mid left(i,jright)in Mtimes Nright]$, and since $T+W$ is a submodule of this $kk$-module, we obtain $kkleft[Z_{i,j}mid left(i,jright)in Mtimes Nright]=T+W$ from eqref{darij.pf.thm1.2}.



Applying Lemma 2 to $C=kkleft[Z_{i,j}mid left(i,jright)in Mtimes Nright]$, $A=T$, $B=W$ and $psi=phi$, we thus conclude that $Ker phi = W$. This proves Theorem 1. $blacksquare$





There is yet another way to prove Theorem 1 -- namely, by revealing it to be a particular case of the Second Fundamental Theorem of Invariant Theory for GL. See https://mathoverflow.net/questions/202005/a-vector-version-of-the-segre-embedding-what-is-the-kernel-of-the-ring-map for this generalization. (Another place where this generalization appears with proof is Theorem 5.1 of J. Désarménien, Joseph P. S. Kung, Gian-Carlo Rota, Invariant Theory, Young Bitableaux, and Combinatorics, unofficial re-edition 2017; you just need to set $d = 1$, and realize that every standard $left(mathcal{X},mathcal{U}right)$-bideterminant of shape strictly longer than $left(dright)$ contains at least one row of length $geq 2$, which is easily seen to place it inside the ideal $W$.)






share|cite|improve this answer



















  • 3




    Dear Darij, thanks for your answer. I posted an answer below showing that we don't need to prove explicitly that the kernel is a prime ideal to show that the image of the Segre embedding is a projective variety. It should be correct. Regards,
    – user38268
    Apr 8 '13 at 5:55










  • Regarding OP's original solution - why is the observation that $operatorname{Im}psi$ equals the vanishing locus of the ideal $I$ generated by the mentioned monomials, along with the observation $I$ is prime not enough?
    – Arrow
    2 days ago










  • @Arrow: how do you prove $I$ is prime?
    – darij grinberg
    2 days ago










  • thanks for this. It's great! BTW, in your definition of $T$ don't you need to use strict inequalities?
    – user347489
    yesterday










  • @user347489: No, I don't. Is anything wrong with the argument?
    – darij grinberg
    yesterday













up vote
9
down vote










up vote
9
down vote









I outlined a proof by Bjorn Poonen at http://www.artofproblemsolving.com/Forum/viewtopic.php?p=2450856#p2450856 . As Nils Matthes has noticed, computing the kernel is not necessary to solve Hartshorne's problem, though (in my opinion) it is more interesting than the problem itself.





$newcommand{kk}{mathbb{k}}$
$newcommand{Ker}{operatorname{Ker}}$
For the sake of self-containedness, let me repost the proof linked above here (with improved notations). I begin by restating the problem:




Theorem 1. Let $kk$ be a commutative ring with $1$. Let $n$ and $m$ be two nonnegative integers. Let $M=left{0,1,ldots ,mright}$ and $N=left{0,1,ldots ,nright}$.



Let $R$ be the polynomial ring $kkleft[Z_{i,j} mid i in M text{ and } j in N right]$.



Let $S$ be the polynomial ring $kkleft[x_0, x_1, ldots, x_m, y_0, y_1, ldots, y_n right]$.



Let $phi : R to S$ be the unique $kk$-algebra homomorphism that sends each $Z_{i,j}$ to $x_i y_j$.



Let $W$ be the ideal of $R$ generated by all elements of the type $Z_{a,b} Z_{c,d} - Z_{a,d} Z_{c,b}$ with $a in M$, $b in N$, $c in M$ and $d in N$.



Then, $Ker phi = W$.




To prove this, we shall use the following easy algebraic lemma:




Lemma 2. Let $C$ be a $kk$-module. Let $A$ and $B$ be two submodules of $C$ such that $C=A+B$. Let $psi$ be a $kk$-module map from $C$ to another $kk$-module $D$ such that $psimid_A$ is injective and $Ker psi supseteq B$. Then, $Ker psi = B$.




Proof of Lemma 2. Let $c in Ker psi$. Thus, $c in Ker psi subseteq C = A + B$; hence, we can write $c$ in the form $c = a + b$ for some $a in A$ and $b in B$. Consider these $a$ and $b$. We have $b in B subseteq Ker psi$, so that $psileft(bright) = 0$. Applying the map $psi$ to the equality $c = a + b$, we obtain $psileft(cright) = psileft(a + bright) = psileft(aright) + psileft(bright)$ (since $psi$ is a $kk$-module map). Comparing this with $psileft(cright) = 0$ (which follows from $c in Ker psi$), we obtain $0 = psileft(aright) + underbrace{psileft(bright)}_{= 0} = psileft(aright)$, so that $psileft(aright) = 0 = psileft(0right)$. Since $psimid_A$ is injective, this entails $a = 0$ (because both $a$ and $0$ belong to $A$). Thus, $c = underbrace{a}_{=0} + b = b in B$.



Now, forget that we fixed $c$. We thus have shown that $c in B$ for each $c in Ker psi$. Thus, $Ker psi subseteq B$. Combining this with $Ker psi supseteq B$, we obtain $Ker psi = B$. This proves Lemma 2. $blacksquare$



Proof of Theorem 1 (Bjorn Poonen) (sketched). We notice that $Ker phisupseteq W$ is very easy to prove (in fact, a trivial computation shows that $Ker phi$ contains $Z_{a,b}Z_{c,d}-Z_{a,d}Z_{c,b}$ for all $a$, $b$, $c$, $d$).



We order the set $Mtimes N$ lexicographically.



If $k$ is a nonnegative integer, then $S_k$ shall denote the symmetric group consisting of all permutations of $left{1,2,ldots,kright}$.
Every $k$-tuple $left(left(a_1,b_1right),left(a_2,b_2right),ldots ,left(a_k,b_kright)right) in left(Mtimes Nright)^k$ and every permutation $sigma in S_k$ satisfy



begin{equation}
Z_{a_1,b_1}Z_{a_2,b_2}cdots Z_{a_k,b_k} equiv Z_{a_1,b_{sigma 1}}Z_{a_2,b_{sigma 2}}cdots Z_{a_k,b_{sigma k}} mod W .
label{darij.pf.thm1.1}
tag{1}
end{equation}



(In fact, this is obvious from the definition of $W$ when $sigma$ is a transposition, and hence, by induction, it also holds for every permutation $sigma$, because every permutation is a composition of transpositions.)



Now, let $T$ be the $kk$-submodule of $kkleft[Z_{i,j}mid left(i,jright)in Mtimes Nright]$ generated by all products of the form $Z_{a_1,c_1}Z_{a_2,c_2}cdots Z_{a_k,c_k}$ with $k$ being a nonnegative integer and $left(left(a_1,c_1right),left(a_2,c_2right),ldots,left(a_k,c_kright)right)in left(Mtimes Nright)^k$ being a $k$-tuple satisfying $a_1leq a_2leq cdotsleq a_k$ and $c_1leq c_2leq cdotsleq c_k$. It is easy to see that the map $left.phimid_Tright. : T to kkleft[x_0,x_1,ldots,x_m,y_0,y_1,ldots,y_nright]$ is injective. (In fact, if $left(left(a_1,c_1right),left(a_2,c_2right),ldots,left(a_k,c_kright)right)in T$, then
begin{align}
left(phimid_Tright)left(Z_{a_1,c_1}Z_{a_2,c_2}cdots Z_{a_k,c_k}right)
&= phileft(Z_{a_1,c_1}Z_{a_2,c_2}cdots Z_{a_k,c_k}right) \
&= x_{a_1}y_{c_1}x_{a_2}y_{c_2}cdots x_{a_k}y_{c_k} \
&=x_{a_1}x_{a_2}cdots x_{a_k}y_{c_1}y_{c_2}cdots y_{c_k}
end{align}

is a monomial from which we can recover the $k$-tuple $left(a_1,a_2,ldots ,a_kright)$ up to order and the $k$-tuple $left(c_1,c_2,ldots ,c_kright)$ up to order; but since the order of each of these two $k$-tuples is predetermined by the condition that $a_1leq a_2leq cdotsleq a_k$ and $c_1leq c_2leq cdotsleq c_k$, we can therefore recover these two $k$-tuples completely; hence, the map $phimid_T$ sends distinct monomials to distinct monomials, and thus is injective.)



Next we are going to show that:



begin{equation}
text{every monomial in $kkleft[Z_{i,j}mid left(i,jright)in Mtimes Nright]$ lies in $T+W$.}
label{darij.pf.thm1.2}
tag{2}
end{equation}



[Proof of eqref{darij.pf.thm1.2}: Let $mu$ be any monomial in $kkleft[Z_{i,j}mid left(i,jright)in Mtimes Nright]$. Then, $mu=Z_{a_1,b_1}Z_{a_2,b_2}cdots Z_{a_k,b_k}$ for some nonnegative integer $k$ and some $k$-tuple $left(left(a_1,b_1right),left(a_2,b_2right),ldots,left(a_k,b_kright)right)in left(Mtimes Nright)^k$ such that $left(a_1,b_1right)leqleft(a_2,b_2right)leq cdotsleq left(a_k,b_kright)$. Consider such a $k$ and such a $k$-tuple $left(left(a_1,b_1right),left(a_2,b_2right),ldots,left(a_k,b_kright)right)$. Since $left(a_1,b_1right)leqleft(a_2,b_2right)leq cdotsleq left(a_k,b_kright)$, we have $a_1leq a_2leq cdotsleq a_k$ (since our order is lexicographic). Clearly there exists a permutation $sigmain S_k$ such that $b_{sigma 1}leq b_{sigma 2}leq cdots leq b_{sigma k}$. Consider such a $sigma$. Let $c_i=b_{sigma i}$ for every $iinleft{1,2,ldots,kright}$. Hence, the chain of inequalities $b_{sigma 1}leq b_{sigma 2}leq cdots leq b_{sigma k}$ rewrites as $c_1leq c_2leq cdotsleq c_k$. Also,
begin{align}
mu
&= Z_{a_1,b_1}Z_{a_2,b_2}cdots Z_{a_k,b_k} \
&equiv Z_{a_1,b_{sigma 1}}Z_{a_2,b_{sigma 2}}cdots Z_{a_k,b_{sigma k}}
qquad left( text{by eqref{darij.pf.thm1.1}} right) \
&= Z_{a_1,c_1} Z_{a_2,c_2} cdots Z_{a_k,c_k} mod W
end{align}

(since $b_{sigma i}=c_i$ for every $iinleft{1,2,ldots,kright}$).



But since $a_1leq a_2leq cdotsleq a_k$ and $c_1leq c_2leq cdotsleq c_k$, we have $Z_{a_1,c_1} Z_{a_2,c_2} cdots Z_{a_k,c_k}in T$ (by the definition of $T$), so this rewrites as follows:
begin{align}
mu
&equiv left(text{an element of }Tright)mod W .
end{align}

In other words, $muin T+W$. Since this holds for every monomial $mu$ in $kkleft[Z_{i,j}mid left(i,jright)in Mtimes Nright]$, this proves eqref{darij.pf.thm1.2}.]



Since the monomials in $kkleft[Z_{i,j}mid left(i,jright)in Mtimes Nright]$ generate the $kk$-module $kkleft[Z_{i,j}mid left(i,jright)in Mtimes Nright]$, and since $T+W$ is a submodule of this $kk$-module, we obtain $kkleft[Z_{i,j}mid left(i,jright)in Mtimes Nright]=T+W$ from eqref{darij.pf.thm1.2}.



Applying Lemma 2 to $C=kkleft[Z_{i,j}mid left(i,jright)in Mtimes Nright]$, $A=T$, $B=W$ and $psi=phi$, we thus conclude that $Ker phi = W$. This proves Theorem 1. $blacksquare$





There is yet another way to prove Theorem 1 -- namely, by revealing it to be a particular case of the Second Fundamental Theorem of Invariant Theory for GL. See https://mathoverflow.net/questions/202005/a-vector-version-of-the-segre-embedding-what-is-the-kernel-of-the-ring-map for this generalization. (Another place where this generalization appears with proof is Theorem 5.1 of J. Désarménien, Joseph P. S. Kung, Gian-Carlo Rota, Invariant Theory, Young Bitableaux, and Combinatorics, unofficial re-edition 2017; you just need to set $d = 1$, and realize that every standard $left(mathcal{X},mathcal{U}right)$-bideterminant of shape strictly longer than $left(dright)$ contains at least one row of length $geq 2$, which is easily seen to place it inside the ideal $W$.)






share|cite|improve this answer














I outlined a proof by Bjorn Poonen at http://www.artofproblemsolving.com/Forum/viewtopic.php?p=2450856#p2450856 . As Nils Matthes has noticed, computing the kernel is not necessary to solve Hartshorne's problem, though (in my opinion) it is more interesting than the problem itself.





$newcommand{kk}{mathbb{k}}$
$newcommand{Ker}{operatorname{Ker}}$
For the sake of self-containedness, let me repost the proof linked above here (with improved notations). I begin by restating the problem:




Theorem 1. Let $kk$ be a commutative ring with $1$. Let $n$ and $m$ be two nonnegative integers. Let $M=left{0,1,ldots ,mright}$ and $N=left{0,1,ldots ,nright}$.



Let $R$ be the polynomial ring $kkleft[Z_{i,j} mid i in M text{ and } j in N right]$.



Let $S$ be the polynomial ring $kkleft[x_0, x_1, ldots, x_m, y_0, y_1, ldots, y_n right]$.



Let $phi : R to S$ be the unique $kk$-algebra homomorphism that sends each $Z_{i,j}$ to $x_i y_j$.



Let $W$ be the ideal of $R$ generated by all elements of the type $Z_{a,b} Z_{c,d} - Z_{a,d} Z_{c,b}$ with $a in M$, $b in N$, $c in M$ and $d in N$.



Then, $Ker phi = W$.




To prove this, we shall use the following easy algebraic lemma:




Lemma 2. Let $C$ be a $kk$-module. Let $A$ and $B$ be two submodules of $C$ such that $C=A+B$. Let $psi$ be a $kk$-module map from $C$ to another $kk$-module $D$ such that $psimid_A$ is injective and $Ker psi supseteq B$. Then, $Ker psi = B$.




Proof of Lemma 2. Let $c in Ker psi$. Thus, $c in Ker psi subseteq C = A + B$; hence, we can write $c$ in the form $c = a + b$ for some $a in A$ and $b in B$. Consider these $a$ and $b$. We have $b in B subseteq Ker psi$, so that $psileft(bright) = 0$. Applying the map $psi$ to the equality $c = a + b$, we obtain $psileft(cright) = psileft(a + bright) = psileft(aright) + psileft(bright)$ (since $psi$ is a $kk$-module map). Comparing this with $psileft(cright) = 0$ (which follows from $c in Ker psi$), we obtain $0 = psileft(aright) + underbrace{psileft(bright)}_{= 0} = psileft(aright)$, so that $psileft(aright) = 0 = psileft(0right)$. Since $psimid_A$ is injective, this entails $a = 0$ (because both $a$ and $0$ belong to $A$). Thus, $c = underbrace{a}_{=0} + b = b in B$.



Now, forget that we fixed $c$. We thus have shown that $c in B$ for each $c in Ker psi$. Thus, $Ker psi subseteq B$. Combining this with $Ker psi supseteq B$, we obtain $Ker psi = B$. This proves Lemma 2. $blacksquare$



Proof of Theorem 1 (Bjorn Poonen) (sketched). We notice that $Ker phisupseteq W$ is very easy to prove (in fact, a trivial computation shows that $Ker phi$ contains $Z_{a,b}Z_{c,d}-Z_{a,d}Z_{c,b}$ for all $a$, $b$, $c$, $d$).



We order the set $Mtimes N$ lexicographically.



If $k$ is a nonnegative integer, then $S_k$ shall denote the symmetric group consisting of all permutations of $left{1,2,ldots,kright}$.
Every $k$-tuple $left(left(a_1,b_1right),left(a_2,b_2right),ldots ,left(a_k,b_kright)right) in left(Mtimes Nright)^k$ and every permutation $sigma in S_k$ satisfy



begin{equation}
Z_{a_1,b_1}Z_{a_2,b_2}cdots Z_{a_k,b_k} equiv Z_{a_1,b_{sigma 1}}Z_{a_2,b_{sigma 2}}cdots Z_{a_k,b_{sigma k}} mod W .
label{darij.pf.thm1.1}
tag{1}
end{equation}



(In fact, this is obvious from the definition of $W$ when $sigma$ is a transposition, and hence, by induction, it also holds for every permutation $sigma$, because every permutation is a composition of transpositions.)



Now, let $T$ be the $kk$-submodule of $kkleft[Z_{i,j}mid left(i,jright)in Mtimes Nright]$ generated by all products of the form $Z_{a_1,c_1}Z_{a_2,c_2}cdots Z_{a_k,c_k}$ with $k$ being a nonnegative integer and $left(left(a_1,c_1right),left(a_2,c_2right),ldots,left(a_k,c_kright)right)in left(Mtimes Nright)^k$ being a $k$-tuple satisfying $a_1leq a_2leq cdotsleq a_k$ and $c_1leq c_2leq cdotsleq c_k$. It is easy to see that the map $left.phimid_Tright. : T to kkleft[x_0,x_1,ldots,x_m,y_0,y_1,ldots,y_nright]$ is injective. (In fact, if $left(left(a_1,c_1right),left(a_2,c_2right),ldots,left(a_k,c_kright)right)in T$, then
begin{align}
left(phimid_Tright)left(Z_{a_1,c_1}Z_{a_2,c_2}cdots Z_{a_k,c_k}right)
&= phileft(Z_{a_1,c_1}Z_{a_2,c_2}cdots Z_{a_k,c_k}right) \
&= x_{a_1}y_{c_1}x_{a_2}y_{c_2}cdots x_{a_k}y_{c_k} \
&=x_{a_1}x_{a_2}cdots x_{a_k}y_{c_1}y_{c_2}cdots y_{c_k}
end{align}

is a monomial from which we can recover the $k$-tuple $left(a_1,a_2,ldots ,a_kright)$ up to order and the $k$-tuple $left(c_1,c_2,ldots ,c_kright)$ up to order; but since the order of each of these two $k$-tuples is predetermined by the condition that $a_1leq a_2leq cdotsleq a_k$ and $c_1leq c_2leq cdotsleq c_k$, we can therefore recover these two $k$-tuples completely; hence, the map $phimid_T$ sends distinct monomials to distinct monomials, and thus is injective.)



Next we are going to show that:



begin{equation}
text{every monomial in $kkleft[Z_{i,j}mid left(i,jright)in Mtimes Nright]$ lies in $T+W$.}
label{darij.pf.thm1.2}
tag{2}
end{equation}



[Proof of eqref{darij.pf.thm1.2}: Let $mu$ be any monomial in $kkleft[Z_{i,j}mid left(i,jright)in Mtimes Nright]$. Then, $mu=Z_{a_1,b_1}Z_{a_2,b_2}cdots Z_{a_k,b_k}$ for some nonnegative integer $k$ and some $k$-tuple $left(left(a_1,b_1right),left(a_2,b_2right),ldots,left(a_k,b_kright)right)in left(Mtimes Nright)^k$ such that $left(a_1,b_1right)leqleft(a_2,b_2right)leq cdotsleq left(a_k,b_kright)$. Consider such a $k$ and such a $k$-tuple $left(left(a_1,b_1right),left(a_2,b_2right),ldots,left(a_k,b_kright)right)$. Since $left(a_1,b_1right)leqleft(a_2,b_2right)leq cdotsleq left(a_k,b_kright)$, we have $a_1leq a_2leq cdotsleq a_k$ (since our order is lexicographic). Clearly there exists a permutation $sigmain S_k$ such that $b_{sigma 1}leq b_{sigma 2}leq cdots leq b_{sigma k}$. Consider such a $sigma$. Let $c_i=b_{sigma i}$ for every $iinleft{1,2,ldots,kright}$. Hence, the chain of inequalities $b_{sigma 1}leq b_{sigma 2}leq cdots leq b_{sigma k}$ rewrites as $c_1leq c_2leq cdotsleq c_k$. Also,
begin{align}
mu
&= Z_{a_1,b_1}Z_{a_2,b_2}cdots Z_{a_k,b_k} \
&equiv Z_{a_1,b_{sigma 1}}Z_{a_2,b_{sigma 2}}cdots Z_{a_k,b_{sigma k}}
qquad left( text{by eqref{darij.pf.thm1.1}} right) \
&= Z_{a_1,c_1} Z_{a_2,c_2} cdots Z_{a_k,c_k} mod W
end{align}

(since $b_{sigma i}=c_i$ for every $iinleft{1,2,ldots,kright}$).



But since $a_1leq a_2leq cdotsleq a_k$ and $c_1leq c_2leq cdotsleq c_k$, we have $Z_{a_1,c_1} Z_{a_2,c_2} cdots Z_{a_k,c_k}in T$ (by the definition of $T$), so this rewrites as follows:
begin{align}
mu
&equiv left(text{an element of }Tright)mod W .
end{align}

In other words, $muin T+W$. Since this holds for every monomial $mu$ in $kkleft[Z_{i,j}mid left(i,jright)in Mtimes Nright]$, this proves eqref{darij.pf.thm1.2}.]



Since the monomials in $kkleft[Z_{i,j}mid left(i,jright)in Mtimes Nright]$ generate the $kk$-module $kkleft[Z_{i,j}mid left(i,jright)in Mtimes Nright]$, and since $T+W$ is a submodule of this $kk$-module, we obtain $kkleft[Z_{i,j}mid left(i,jright)in Mtimes Nright]=T+W$ from eqref{darij.pf.thm1.2}.



Applying Lemma 2 to $C=kkleft[Z_{i,j}mid left(i,jright)in Mtimes Nright]$, $A=T$, $B=W$ and $psi=phi$, we thus conclude that $Ker phi = W$. This proves Theorem 1. $blacksquare$





There is yet another way to prove Theorem 1 -- namely, by revealing it to be a particular case of the Second Fundamental Theorem of Invariant Theory for GL. See https://mathoverflow.net/questions/202005/a-vector-version-of-the-segre-embedding-what-is-the-kernel-of-the-ring-map for this generalization. (Another place where this generalization appears with proof is Theorem 5.1 of J. Désarménien, Joseph P. S. Kung, Gian-Carlo Rota, Invariant Theory, Young Bitableaux, and Combinatorics, unofficial re-edition 2017; you just need to set $d = 1$, and realize that every standard $left(mathcal{X},mathcal{U}right)$-bideterminant of shape strictly longer than $left(dright)$ contains at least one row of length $geq 2$, which is easily seen to place it inside the ideal $W$.)







share|cite|improve this answer














share|cite|improve this answer



share|cite|improve this answer








edited yesterday

























answered Apr 8 '13 at 5:42









darij grinberg

9,81032961




9,81032961








  • 3




    Dear Darij, thanks for your answer. I posted an answer below showing that we don't need to prove explicitly that the kernel is a prime ideal to show that the image of the Segre embedding is a projective variety. It should be correct. Regards,
    – user38268
    Apr 8 '13 at 5:55










  • Regarding OP's original solution - why is the observation that $operatorname{Im}psi$ equals the vanishing locus of the ideal $I$ generated by the mentioned monomials, along with the observation $I$ is prime not enough?
    – Arrow
    2 days ago










  • @Arrow: how do you prove $I$ is prime?
    – darij grinberg
    2 days ago










  • thanks for this. It's great! BTW, in your definition of $T$ don't you need to use strict inequalities?
    – user347489
    yesterday










  • @user347489: No, I don't. Is anything wrong with the argument?
    – darij grinberg
    yesterday














  • 3




    Dear Darij, thanks for your answer. I posted an answer below showing that we don't need to prove explicitly that the kernel is a prime ideal to show that the image of the Segre embedding is a projective variety. It should be correct. Regards,
    – user38268
    Apr 8 '13 at 5:55










  • Regarding OP's original solution - why is the observation that $operatorname{Im}psi$ equals the vanishing locus of the ideal $I$ generated by the mentioned monomials, along with the observation $I$ is prime not enough?
    – Arrow
    2 days ago










  • @Arrow: how do you prove $I$ is prime?
    – darij grinberg
    2 days ago










  • thanks for this. It's great! BTW, in your definition of $T$ don't you need to use strict inequalities?
    – user347489
    yesterday










  • @user347489: No, I don't. Is anything wrong with the argument?
    – darij grinberg
    yesterday








3




3




Dear Darij, thanks for your answer. I posted an answer below showing that we don't need to prove explicitly that the kernel is a prime ideal to show that the image of the Segre embedding is a projective variety. It should be correct. Regards,
– user38268
Apr 8 '13 at 5:55




Dear Darij, thanks for your answer. I posted an answer below showing that we don't need to prove explicitly that the kernel is a prime ideal to show that the image of the Segre embedding is a projective variety. It should be correct. Regards,
– user38268
Apr 8 '13 at 5:55












Regarding OP's original solution - why is the observation that $operatorname{Im}psi$ equals the vanishing locus of the ideal $I$ generated by the mentioned monomials, along with the observation $I$ is prime not enough?
– Arrow
2 days ago




Regarding OP's original solution - why is the observation that $operatorname{Im}psi$ equals the vanishing locus of the ideal $I$ generated by the mentioned monomials, along with the observation $I$ is prime not enough?
– Arrow
2 days ago












@Arrow: how do you prove $I$ is prime?
– darij grinberg
2 days ago




@Arrow: how do you prove $I$ is prime?
– darij grinberg
2 days ago












thanks for this. It's great! BTW, in your definition of $T$ don't you need to use strict inequalities?
– user347489
yesterday




thanks for this. It's great! BTW, in your definition of $T$ don't you need to use strict inequalities?
– user347489
yesterday












@user347489: No, I don't. Is anything wrong with the argument?
– darij grinberg
yesterday




@user347489: No, I don't. Is anything wrong with the argument?
– darij grinberg
yesterday










up vote
6
down vote



accepted










Indeed as Nils Matthes suggested we don't know need to go through such a mess and just use the hint of Hartshorne.



Define a map $varphi : k[T_{00} , ldots, T_{nm}] to k[X_0,ldots,X_n,Y_0,ldots,Y_m]$ that sends $T_{ij}$ to $X_iY_j$. Let $mathfrak{a} := ker varphi$. We claim that $psi (Bbb{P}^n times Bbb{P}^m) = V(mathfrak{a})$. For one inclusion if a point
$$a = [a_{00} : ldots : a_{nm}] in V(mathfrak{a})$$
then in particular $a$ is a zero of all the polynomials $T_{ij}T_{kl} - T_{il}T_{kj}$. But this means that $a in psi(Bbb{P}^n times Bbb{P}^m)$. The reverse inclusion follows immediately from the definition of $varphi$. Thus $V(mathfrak{a}) = psi(Bbb{P}^n times Bbb{P}^m)$ and the projective Nullstellensatz implies that $psi(Bbb{P}^n times Bbb{P}^m)$ is a projective variety.






share|cite|improve this answer



















  • 1




    Shouldn't you prove that $mathfrak a$ is actually a homogeneous ideal?
    – Andrea Gagna
    May 15 '14 at 15:46















up vote
6
down vote



accepted










Indeed as Nils Matthes suggested we don't know need to go through such a mess and just use the hint of Hartshorne.



Define a map $varphi : k[T_{00} , ldots, T_{nm}] to k[X_0,ldots,X_n,Y_0,ldots,Y_m]$ that sends $T_{ij}$ to $X_iY_j$. Let $mathfrak{a} := ker varphi$. We claim that $psi (Bbb{P}^n times Bbb{P}^m) = V(mathfrak{a})$. For one inclusion if a point
$$a = [a_{00} : ldots : a_{nm}] in V(mathfrak{a})$$
then in particular $a$ is a zero of all the polynomials $T_{ij}T_{kl} - T_{il}T_{kj}$. But this means that $a in psi(Bbb{P}^n times Bbb{P}^m)$. The reverse inclusion follows immediately from the definition of $varphi$. Thus $V(mathfrak{a}) = psi(Bbb{P}^n times Bbb{P}^m)$ and the projective Nullstellensatz implies that $psi(Bbb{P}^n times Bbb{P}^m)$ is a projective variety.






share|cite|improve this answer



















  • 1




    Shouldn't you prove that $mathfrak a$ is actually a homogeneous ideal?
    – Andrea Gagna
    May 15 '14 at 15:46













up vote
6
down vote



accepted







up vote
6
down vote



accepted






Indeed as Nils Matthes suggested we don't know need to go through such a mess and just use the hint of Hartshorne.



Define a map $varphi : k[T_{00} , ldots, T_{nm}] to k[X_0,ldots,X_n,Y_0,ldots,Y_m]$ that sends $T_{ij}$ to $X_iY_j$. Let $mathfrak{a} := ker varphi$. We claim that $psi (Bbb{P}^n times Bbb{P}^m) = V(mathfrak{a})$. For one inclusion if a point
$$a = [a_{00} : ldots : a_{nm}] in V(mathfrak{a})$$
then in particular $a$ is a zero of all the polynomials $T_{ij}T_{kl} - T_{il}T_{kj}$. But this means that $a in psi(Bbb{P}^n times Bbb{P}^m)$. The reverse inclusion follows immediately from the definition of $varphi$. Thus $V(mathfrak{a}) = psi(Bbb{P}^n times Bbb{P}^m)$ and the projective Nullstellensatz implies that $psi(Bbb{P}^n times Bbb{P}^m)$ is a projective variety.






share|cite|improve this answer














Indeed as Nils Matthes suggested we don't know need to go through such a mess and just use the hint of Hartshorne.



Define a map $varphi : k[T_{00} , ldots, T_{nm}] to k[X_0,ldots,X_n,Y_0,ldots,Y_m]$ that sends $T_{ij}$ to $X_iY_j$. Let $mathfrak{a} := ker varphi$. We claim that $psi (Bbb{P}^n times Bbb{P}^m) = V(mathfrak{a})$. For one inclusion if a point
$$a = [a_{00} : ldots : a_{nm}] in V(mathfrak{a})$$
then in particular $a$ is a zero of all the polynomials $T_{ij}T_{kl} - T_{il}T_{kj}$. But this means that $a in psi(Bbb{P}^n times Bbb{P}^m)$. The reverse inclusion follows immediately from the definition of $varphi$. Thus $V(mathfrak{a}) = psi(Bbb{P}^n times Bbb{P}^m)$ and the projective Nullstellensatz implies that $psi(Bbb{P}^n times Bbb{P}^m)$ is a projective variety.







share|cite|improve this answer














share|cite|improve this answer



share|cite|improve this answer








edited Oct 30 '14 at 10:16









user26857

39.1k123882




39.1k123882










answered Apr 8 '13 at 5:22







user38268















  • 1




    Shouldn't you prove that $mathfrak a$ is actually a homogeneous ideal?
    – Andrea Gagna
    May 15 '14 at 15:46














  • 1




    Shouldn't you prove that $mathfrak a$ is actually a homogeneous ideal?
    – Andrea Gagna
    May 15 '14 at 15:46








1




1




Shouldn't you prove that $mathfrak a$ is actually a homogeneous ideal?
– Andrea Gagna
May 15 '14 at 15:46




Shouldn't you prove that $mathfrak a$ is actually a homogeneous ideal?
– Andrea Gagna
May 15 '14 at 15:46










up vote
4
down vote













It's easy to show the preimage under the Segre map of an algebraic set is an algebraic subset of $mathbb{P}^n times mathbb{P}^m.$ If $V$ were reducible, we could write $V = V_1 cup V_2,$ where $V_ineq V$ are closed. Then $$ mathbb{P}^n times mathbb{P}^m = S^{-1}(V) = S^{-1} (V_1) bigcup S^{-1}(V_2)$$ where $S^{-1}(V_i)$ are closed. Picking $x_i in Vsetminus V_i$ we have $S^{-1}(x_i)cap S^{-1}(V_i)=emptyset$ so $S^{-1}(V_i) subset S^{-1}(V)$ is a strict inclusion, contradicting that $mathbb{P}^n times mathbb{P}^m$ is irreducible.






share|cite|improve this answer

















  • 3




    Of course we do, it's the same condition as always in any topological space: A space $X$ is reducible if $X= X_1 cup X_2$ for some closed proper subsets $X_1, X_2$ and irreducible if not reducible. A subset $V$ of $mathbb{P}^n times mathbb{P}^m$ is closed if $V=V(S)$ where $S$ is a set of biforms in $k[X_0, cdots, X_n, Y_0, cdots, Y_m].$ This already establishes what it means for $mathbb{P}^n times mathbb{P}^m$ to be irreducible.
    – Ragib Zaman
    Jun 3 '13 at 10:11















up vote
4
down vote













It's easy to show the preimage under the Segre map of an algebraic set is an algebraic subset of $mathbb{P}^n times mathbb{P}^m.$ If $V$ were reducible, we could write $V = V_1 cup V_2,$ where $V_ineq V$ are closed. Then $$ mathbb{P}^n times mathbb{P}^m = S^{-1}(V) = S^{-1} (V_1) bigcup S^{-1}(V_2)$$ where $S^{-1}(V_i)$ are closed. Picking $x_i in Vsetminus V_i$ we have $S^{-1}(x_i)cap S^{-1}(V_i)=emptyset$ so $S^{-1}(V_i) subset S^{-1}(V)$ is a strict inclusion, contradicting that $mathbb{P}^n times mathbb{P}^m$ is irreducible.






share|cite|improve this answer

















  • 3




    Of course we do, it's the same condition as always in any topological space: A space $X$ is reducible if $X= X_1 cup X_2$ for some closed proper subsets $X_1, X_2$ and irreducible if not reducible. A subset $V$ of $mathbb{P}^n times mathbb{P}^m$ is closed if $V=V(S)$ where $S$ is a set of biforms in $k[X_0, cdots, X_n, Y_0, cdots, Y_m].$ This already establishes what it means for $mathbb{P}^n times mathbb{P}^m$ to be irreducible.
    – Ragib Zaman
    Jun 3 '13 at 10:11













up vote
4
down vote










up vote
4
down vote









It's easy to show the preimage under the Segre map of an algebraic set is an algebraic subset of $mathbb{P}^n times mathbb{P}^m.$ If $V$ were reducible, we could write $V = V_1 cup V_2,$ where $V_ineq V$ are closed. Then $$ mathbb{P}^n times mathbb{P}^m = S^{-1}(V) = S^{-1} (V_1) bigcup S^{-1}(V_2)$$ where $S^{-1}(V_i)$ are closed. Picking $x_i in Vsetminus V_i$ we have $S^{-1}(x_i)cap S^{-1}(V_i)=emptyset$ so $S^{-1}(V_i) subset S^{-1}(V)$ is a strict inclusion, contradicting that $mathbb{P}^n times mathbb{P}^m$ is irreducible.






share|cite|improve this answer












It's easy to show the preimage under the Segre map of an algebraic set is an algebraic subset of $mathbb{P}^n times mathbb{P}^m.$ If $V$ were reducible, we could write $V = V_1 cup V_2,$ where $V_ineq V$ are closed. Then $$ mathbb{P}^n times mathbb{P}^m = S^{-1}(V) = S^{-1} (V_1) bigcup S^{-1}(V_2)$$ where $S^{-1}(V_i)$ are closed. Picking $x_i in Vsetminus V_i$ we have $S^{-1}(x_i)cap S^{-1}(V_i)=emptyset$ so $S^{-1}(V_i) subset S^{-1}(V)$ is a strict inclusion, contradicting that $mathbb{P}^n times mathbb{P}^m$ is irreducible.







share|cite|improve this answer












share|cite|improve this answer



share|cite|improve this answer










answered Jun 3 '13 at 9:20









Ragib Zaman

28.3k34889




28.3k34889








  • 3




    Of course we do, it's the same condition as always in any topological space: A space $X$ is reducible if $X= X_1 cup X_2$ for some closed proper subsets $X_1, X_2$ and irreducible if not reducible. A subset $V$ of $mathbb{P}^n times mathbb{P}^m$ is closed if $V=V(S)$ where $S$ is a set of biforms in $k[X_0, cdots, X_n, Y_0, cdots, Y_m].$ This already establishes what it means for $mathbb{P}^n times mathbb{P}^m$ to be irreducible.
    – Ragib Zaman
    Jun 3 '13 at 10:11














  • 3




    Of course we do, it's the same condition as always in any topological space: A space $X$ is reducible if $X= X_1 cup X_2$ for some closed proper subsets $X_1, X_2$ and irreducible if not reducible. A subset $V$ of $mathbb{P}^n times mathbb{P}^m$ is closed if $V=V(S)$ where $S$ is a set of biforms in $k[X_0, cdots, X_n, Y_0, cdots, Y_m].$ This already establishes what it means for $mathbb{P}^n times mathbb{P}^m$ to be irreducible.
    – Ragib Zaman
    Jun 3 '13 at 10:11








3




3




Of course we do, it's the same condition as always in any topological space: A space $X$ is reducible if $X= X_1 cup X_2$ for some closed proper subsets $X_1, X_2$ and irreducible if not reducible. A subset $V$ of $mathbb{P}^n times mathbb{P}^m$ is closed if $V=V(S)$ where $S$ is a set of biforms in $k[X_0, cdots, X_n, Y_0, cdots, Y_m].$ This already establishes what it means for $mathbb{P}^n times mathbb{P}^m$ to be irreducible.
– Ragib Zaman
Jun 3 '13 at 10:11




Of course we do, it's the same condition as always in any topological space: A space $X$ is reducible if $X= X_1 cup X_2$ for some closed proper subsets $X_1, X_2$ and irreducible if not reducible. A subset $V$ of $mathbb{P}^n times mathbb{P}^m$ is closed if $V=V(S)$ where $S$ is a set of biforms in $k[X_0, cdots, X_n, Y_0, cdots, Y_m].$ This already establishes what it means for $mathbb{P}^n times mathbb{P}^m$ to be irreducible.
– Ragib Zaman
Jun 3 '13 at 10:11


















 

draft saved


draft discarded



















































 


draft saved


draft discarded














StackExchange.ready(
function () {
StackExchange.openid.initPostLogin('.new-post-login', 'https%3a%2f%2fmath.stackexchange.com%2fquestions%2f353846%2fhartshorne-problem-1-2-14-on-segre-embedding%23new-answer', 'question_page');
}
);

Post as a guest















Required, but never shown





















































Required, but never shown














Required, but never shown












Required, but never shown







Required, but never shown

































Required, but never shown














Required, but never shown












Required, but never shown







Required, but never shown







Popular posts from this blog

Can a sorcerer learn a 5th-level spell early by creating spell slots using the Font of Magic feature?

Does disintegrating a polymorphed enemy still kill it after the 2018 errata?

A Topological Invariant for $pi_3(U(n))$